Game Setup?
Can you please post the game set up for these questions?
MGN2014 on September 26, 2014
  • June 2009 LSAT
  • SEC1
  • Q13
2
Replies
Can you please clarify?
The correct answer choice is C, which states that the reasoning fails to consider that patients t...
NicoCapri on September 26, 2014
  • September 2007 LSAT
  • SEC3
  • Q16
1
Reply
Additional Question
I picked (A) over (E) after pondering over choosing between both. But, I still don't understand w...
Batman on September 26, 2014
  • October 1997 LSAT
  • SEC3
  • Q22
2
Replies
Help!
Can you explain how to answer this question please? Thank you!
Asnodgrass on September 26, 2014
  • September 2009 LSAT
  • SEC4
  • Q20
1
Reply
Confused about answer
I am struggling to understand why B is the answer and D is not. Could you help explain?
krthom on September 25, 2014
  • June 2013 LSAT
  • SEC3
  • Q6
1
Reply
Question
Why is answer b correct and d incorrect?
Eunbeezy on September 25, 2014
  • June 2002 LSAT
  • SEC1
  • Q21
1
Reply
C versus E?
I got this correct, but took far too long deciding between answer choice C and E. C conforms to...
NicoCapri on September 25, 2014
  • December 2006 LSAT
  • SEC1
  • Q17
1
Reply
No question stem
There is no question stem offered
Derek on September 24, 2014
  • October 2012 LSAT
  • SEC4
  • Q13
1
Reply
Clarification
The correct answer states that the argument proceeds by "appealing to a historical fact to suppor...
NicoCapri on September 24, 2014
  • October 2013 LSAT
  • SEC4
  • Q26
1
Reply
Problem
The Game three prompt remains as I go into game 4.
Jprince on September 23, 2014
  • October 1999 LSAT
  • SEC3
  • Q20
1
Reply
Explanation
Why is B the correct answer? Is there a particular way to set this question up?
tselimovic on September 23, 2014
  • September 1995 LSAT
  • SEC3
  • Q22
1
Reply
Why Is E Right
Hi there, Can you please explain why e is right and and c is wrong? I don't see too much of a di...
Asnodgrass on September 22, 2014
  • December 2008 LSAT
  • SEC2
  • Q20
2
Replies
A not C?
I get why A could be the answer, because it describes an alternate cause for risk reduction in wo...
OffshoreSuge on September 19, 2014
  • October 2008 LSAT
  • SEC1
  • Q7
1
Reply
B Vs E?
I understand how E is correct but I don't understand how B is incorrect? With G in 602 then we st...
MGN2014 on September 19, 2014
  • September 2009 LSAT
  • SEC4
  • Q3
1
Reply
E?!
Plz explain why this is the correct answer. I'm utterly lost.
ArashMash89 on September 18, 2014
  • June 2014 LSAT
  • SEC1
  • Q2
1
Reply
Help
I can't clearly understand the weakness of Helen's justification and why (E) is the answer for th...
Batman on September 18, 2014
  • October 1997 LSAT
  • SEC3
  • Q22
1
Reply
Hey Guys
May I get this one diagramed?
Derek on September 16, 2014
  • October 2010 LSAT
  • SEC5
  • Q25
1
Reply
Confused
How is D even relevant?
MGN2014 on September 16, 2014
  • December 2008 LSAT
  • SEC2
  • Q14
2
Replies
Can You Possibly Post Video Explanation Please?
I have no idea what this games structure is supposed to look like... Is this a multilinear game?
MGN2014 on September 15, 2014
  • October 2008 LSAT
  • SEC4
  • Q20
1
Reply
Help!
Can you please explain why D is the right answer? Thank you!
Asnodgrass on September 11, 2014
  • December 2007 LSAT
  • SEC3
  • Q23
1
Reply